@ : Home > Reasoning > Logical Reasoning > Practice Questions

Logical Reasoning Set 9

Lavina wants to take 4 courses this semester. There are only 7 courses in which she is interested and that do not conflict with her job: three science courses - B, C and P - and four arts courses - L, D, M and H. To meet college requirements she must take two science courses this semester. There are some scheduling problems, however: L overlaps both C and M, which are subsequential; B is given at the same time as D.

Must Read:




Check Your Learning:



Q1) If Lavina deceides to take L, what will her other three courses be?

1) B, P, and C
2) B, P, and H
3) B, P, and D
4) H, P, and C

Q2) If course P is at the same time as L and Lavina takes P, what further problem(s) will she face?

1) She won't be able to take two science classes
2) She won't be able to take B
3) She won't be able to take H
4) She won't be able to take 4 courses which interest her

Q3) If course C is changed to a time which Lavina cannot make and she deceides to take M, then which of the following would be her schedule?

1) B, P, L and M
2) B, P, D and M
3) B, P, H and M
4) H, P, L and M

Q4) If Lavina takes 4 courses this semester, she cannot?

I) Take D and not take C
II) Take M and not take C
III) Take L and not take P
1) I only
2) II only
3) III only
4) I and III only

Q5) Which of the following must always be true?

I) Lavina must take P if she takes M
II) Lavina must take C if she takes D
III) Lavina must take D if she takes C
1) I, II and III
2) II and III only
3) I and II only
4) II only

Solutions and Explanations

1) If Lavina takes L then C and M are out. If C is out, then P and B is scheduled, then D is out. Hence, option 2.

2) When Lavina takes P, now scheduled at the same time as L, she cannot take L, C, or M because they overlap with the new P time. Only D or B may be taken, plus H. Hence, option 4.

3) If C is out then B, P are required. B precludes D and M displaces L. Hence, option 3.

4) III work by the same logic. In each case the taking of one of the non-science courses eliminates one of the science courses from consideration, thus requiring the other two science courses. II is trying to trap you into saying that since neither M nor C can combine with L, they must combine with each other. This is not true. It is possible to have a curriculum of M, P, B and H. Hence, option 4.

5) I must be true because if M is taken then C cnnot be taken, implies the remaining two science subjects must be taken. II must be true since D eliminates B, and requires the other two sciences. III need not be true since a curriculum of C, B, H and M is but one counter-example. Hence, option 3.




Logical Reasoning